The Student Room Group

Scroll to see replies

Reply 80
BlueAngel
The question on geometric series.

To get the common ratio, did you divide the 2 consecutive numbers???
I think it was 9/10 or 10/9 or summit.
What did everyone do here?


9/10 = 0.9 r = 0.9
Reply 81
I think the series question was:

A) PROOF! - yes! I revised that this morning.

B) Sum = 100

c) Difference = 9/10 or 0.9

d) 10/11
(because 10 = X/(1-X)

therefore,

10(1-X) = X

10 - 10X = X

10 = 11X

X = 10/11
Reply 82
MBR50
I think the series question was:

A) PROOF! - yes! I revised that this morning.

B) Sum = 100

c) Difference = 9/10 or 0.9

d) 10/11
(because 10 = X/(1-X)

therefore,

10(1-X) = X

10 - 10X = X

10 = 11X

X = 10/11


correct - well done :smile:
2slick
also, on the graph of C and l, wot points did every1 get for where they crossed?


i got (1, -3) and the other one was somwething like (-4, 12) :confused: , but i cant remember the last one properly

i got those too...ne1 else get them?
Reply 84
MBR50
I think the series question was:

A) PROOF! - yes! I revised that this morning.

B) Sum = 100

c) Difference = 9/10 or 0.9

d) 10/11
(because 10 = X/(1-X)

therefore,

10(1-X) = X

10 - 10X = X

10 = 11X

X = 10/11


i couldnt do A), i hate myself...
MBR50
for the first one 17 + 12 route 2

The second one 0.5 plus something like (8/3) route 2

did no1 get a - here??? sumthing lke 17-12rout2....wot was da q again??
TheWolf
mm i cant remember but i know that for one of them, there are 2 answers, for the other there are 4 answers

4 answers...i think there were only two answers for each of da trig qs
Reply 87
lgs98jonee
did no1 get a - here??? sumthing lke 17-12rout2....wot was da q again??


I said that that in the part you quoted!!!!

lol

Oh and the proof - you either knew it or you didnt i guess.
Reply 88
oh and i think there were only 2 answers for the triq questions as well.

Don't forget 0<x<360

not -360<x<360
Reply 89
lgs98jonee
4 answers...i think there were only two answers for each of da trig qs


im pretty sure that the 2nd one has 4 answers - the tan one?
MBR50
I said that that in the part you quoted!!!!

lol

Oh and the proof - you either knew it or you didnt i guess.

no u said 17+12 route2 i think it is 17-12route2
i got a feeling I'm going to be borderline on this paper or 2-3 marks of the A because i see myself getting 64-66/ 75
i guess that means I'll have to work extra hard for S1... M1 went brilliantly
TheWolf
im pretty sure that the 2nd one has 4 answers - the tan one?

for the first tan one...there were only two answers right??? and for the second one there were only two as well???
Reply 93
lgs98jonee
no u said 17+12 route2 i think it is 17-12route2


oh right, yeah probably, minor detail :smile:

I messed up mechanics lol. and I will get about 50% in stats. I reckon stats is gonna be hard, esp. after these papers
Reply 94
Ollie
How did P1 go?

Which questions did you find hard/easy?

Are you going to repeat it next year?


it was a nice paper except from the last question..i kinda got confused! sayin this, what did every1 get for x-coordinate of Q??i got 11/3 and 4 for gradient that showed P and Q were parallel..!??!!any1 agree??
Reply 95
erm, i didnt thing P & Q were paralell!?! I thought you had to prove line PQ was paralell to the X axies?
Q's coordinate was def 3 i am afraid

u got 3x^2-14x+15 after differentiating
and so this was (3x-5)(x-3)=0

so x=5/3 (other turning point which u could show using 2nd derivative) or 3

pls correct me if i am wrong
lgs98jonee
for the first tan one...there were only two answers right??? and for the second one there were only two as well???

so did every1 get two answers for each of the trig qs????????
Reply 98
lgs98jonee
so did every1 get two answers for each of the trig qs????????


can you remember the calulator value for the second one?
Reply 99
i think Q was somethin like (3, 12) and P(which was given) was (1,12). for the proof of it being parallel, i wrote that they y values are the same therefore it must be parallel to the x axis, but friends were saying that you had 2 state the gradient of it was 0, but i didnt write this down was i tht it was 2 obvious!! do you guys think i will be deducted for this? :confused:

Latest